Difference between revisions of "2016 AMC 12A Problems/Problem 3"

(Problem)
(Solution)
Line 15: Line 15:
 
<cmath>=\frac{3}{8}-\frac{2}{5}</cmath>
 
<cmath>=\frac{3}{8}-\frac{2}{5}</cmath>
 
<cmath>=\boxed{\textbf{(B)}-\frac{1}{40}}</cmath>
 
<cmath>=\boxed{\textbf{(B)}-\frac{1}{40}}</cmath>
 +
 +
==See Also==
 +
{{AMC12 box|year=2016|ab=A|num-b=2|num-a=4}}
 +
{{MAA Notice}}

Revision as of 23:46, 3 February 2016


Problem

The remainder function can be defined for all real numbers $x$ and $y$ with $y\ne 0$ by $\text{rem}(x,y)=x-y\Big\lfloor\frac{x}{y}\Big\rfloor$, where $\Big\lfloor\frac{x}{y}\Big\rfloor$ denotes the greatest integer less than or equal to $\frac{x}{y}$. What is the value of $\text{rem}\left(\frac{3}{8},-\frac{2}{5}\right)$?

$\textbf{(A)}\ -\frac{3}{8}\qquad\textbf{(B)}\ -\frac{1}{40}\qquad\textbf{(C)}\ 0\qquad\textbf{(D)}\ \frac{3}{8}\qquad\textbf{(E)}\ \frac{31}{40}$

Solution

\[\text{rem}\left(\frac{3}{8},-\frac{2}{5}\right)\] \[=\frac{3}{8}-\left(-\frac{2}{5}\right)\Big\lfloor\frac{\frac{3}{8}}{-\frac{2}{5}}\Big\rfloor\] \[=\frac{3}{8}+\left(\frac{2}{5}\right)\Big\lfloor -\frac{15}{16}\Big\rfloor\] \[=\frac{3}{8}+\left(\frac{2}{5}\right)\left(-1\right)\] \[=\frac{3}{8}-\frac{2}{5}\] \[=\boxed{\textbf{(B)}-\frac{1}{40}}\]

See Also

2016 AMC 12A (ProblemsAnswer KeyResources)
Preceded by
Problem 2
Followed by
Problem 4
1 2 3 4 5 6 7 8 9 10 11 12 13 14 15 16 17 18 19 20 21 22 23 24 25
All AMC 12 Problems and Solutions

The problems on this page are copyrighted by the Mathematical Association of America's American Mathematics Competitions. AMC logo.png